Econ Test 1

Pataasin ang iyong marka sa homework at exams ngayon gamit ang Quizwiz!

Suppose the demand for peaches decreases. What will happen to producer surplus in the market for peaches? a. It decreases. b. It may increase, decrease, or remain unchanged. c. It increases. d. It remains unchanged.

a. It decreases.

If the price elasticity of supply for a good is equal to infinity, then the a. supply curve is vertical. b. supply curve is horizontal. c. supply curve also has a slope equal to infinity. d. quantity supplied is constant regardless of the price.

b. supply curve is horizontal.

Suppose that Firms A and B each produce high-resolution computer monitors, but Firm A can do so at a lower cost. Cassie and David each want to purchase a high-resolution computer monitor, but David is willing to pay more than Cassie. Which of the following market outcomes is efficient? a. Firm A produces a monitor that David buys. b. Firm B produces a monitor that David buys. c. Firm A produces a monitor that Cassie buys. David does not purchase a monitor. d. Firm B produces a monitor that Cassie buys. David does not purchase a monitor.

a. Firm A produces a monitor that David buys.

The production of methamphetamine (meth) is a social problem in the Midwest. Iowa is considering two potential programs: Operation Methbust would increase the number of sheriffs' deputies to search out and destroy methamphetamine labs. Operation Say No to Meth would increase the training required of public school teachers so that they could better educate students about the health risks of using meth. Assuming that each program were successful, which of the following statements is correct? a. Operation Methbust would reduce the supply of meth; Operation Say No would reduce the demand for meth. b. Operation Methbust would reduce the demand for meth; Operation Say No would reduce the supply of meth. c. Both Operation Methbust and Say No would reduce the supply of meth. d. Both Operation Methbust and Say No would reduce the demand for meth.

a. Operation Methbust would reduce the supply of meth; Operation Say No would reduce the demand for meth.

For a particular good, a 10 percent increase in price causes a 5 percent decrease in quantity demanded. Which of the following statements is most likely applicable to this good? a. The good is a necessity. b. The relevant time horizon is long. c. The market for the good is narrowly defined. d. There are many close substitutes for this good.

a. The good is a necessity.

The demand for salt is inelastic, and the supply of salt is elastic. The demand for caviar is elastic, and the supply of caviar is inelastic. Suppose that a tax of $1 per pound is levied on the sellers of salt, and a tax of $1 per pound is levied on the buyers of caviar. We would expect that most of the burden of these taxes will fall on a. buyers of salt and the sellers of caviar. b. sellers of salt and the sellers of caviar. c. sellers of salt and the buyers of caviar. d. buyers of salt and the buyers of caviar.

a. buyers of salt and the sellers of caviar.

Suppose the cross-price elasticity of demand between hot dogs and mustard is -2.00. This implies that a 20 percent increase in the price of hot dogs will cause the quantity of mustard purchased to a. fall by 40 percent. b. rise by 200 percent. c. rise by 40 percent. d. fall by 200 percent.

a. fall by 40 percent.

Coffee and tea are substitutes. Good weather that sharply increases the coffee bean harvest would a. increase consumer surplus in the market for coffee and decrease producer surplus in the market for tea. b. decrease consumer surplus in the market for coffee and increase producer surplus in the market for tea. c. decrease consumer surplus in the market for coffee and decrease producer surplus in the market for tea. d. increase consumer surplus in the market for coffee and increase producer surplus in the market for tea.

a. increase consumer surplus in the market for coffee and decrease producer surplus in the market for tea

When supply and demand both increase, equilibrium a. price may increase, decrease, or remain unchanged. b. price will decrease. c. quantity may increase, decrease, or remain unchanged. d. price will increase.

a. price may increase, decrease, or remain unchanged.

Studies indicate that the price elasticity of demand for cigarettes is about 0.4. A government policy aimed at reducing smoking changed the price of a pack of cigarettes from $2 to $6. According to the midpoint method, the government policy should have reduced smoking by a. 250%. b. 40%. c. 80%. d. 30%.

b. 40%.

Which of the following statements is correct concerning the burden of a tax imposed on take-out food? a. Sellers bear the entire burden of the tax. b. Buyers and sellers share the burden of the tax. c. We have to know whether it is the buyers or the sellers that are required to pay the tax to the government in order to make this determination. d. Buyers bear the entire burden of the tax.

b. Buyers and sellers share the burden of the tax.

Chad is willing to pay $5.00 to get his first cup of morning latté; he is willing to pay $4.50 for a second cup. He buys his first cup from a vendor selling latté for $3.75 per cup. He returns to that vendor later in the morning to find that the vendor has increased her price to $3.90 per cup. Chad buys a second cup. Which of the following statements is correct? a. Chad's consumer surplus on his second cup of latté was larger than his consumer surplus on his first cup of latté. b. Chad's willingness to pay for his second cup of latté was smaller than his willingness to pay for his first cup of latté. c. Chad is irrational in that he is willing to pay a different price for his second cup of latté than what he is willing to pay for his first cup of latté. d. Chad places a higher value on his second cup of latté than on his first cup of latté.

b. Chad's willingness to pay for his second cup of latté was smaller than his willingness to pay for his first cup of latté.

Which of the following statements does not help to explain why government drug interdiction increases drug-related crime? a. The demand for illegal drugs is inelastic. b. Government drug programs are more lenient now with drug offenders than they were in the 1980s. c. Interdiction results in drug addicts having a greater need for quick cash. d. Interdiction results in an increase in the amount of money needed to buy the same amount of drugs.

b. Government drug programs are more lenient now with drug offenders than they were in the 1980s.

Suppose the government has imposed a price ceiling on laptop computers. Which of the following events could transform the price ceiling from one that is not binding into one that is binding? a. Improvements in production technology reduce the costs of producing laptop computers. b. The number of firms selling laptop computers decreases. c. The number of consumers buying laptop computers decreases. d. Consumers' income decreases, and laptop computers are a normal good.

b. The number of firms selling laptop computers decreases.

Which of the following will cause an increase in consumer surplus? a. an increase in the production cost of the good b. a technological improvement in the production of the good c. the imposition of a binding price floor in the market d. a decrease in the number of sellers of the good

b. a technological improvement in the production of the good

Which of the following would not shift the supply curve for mp3 players? a. a decrease in the number of sellers of mp3 players b. an increase in the price of mp3 players c. an increase in the price of plastic, an input into the production of mp3 players d. an improvement in the technology used to produce mp3 players

b. an increase in the price of mp3 players

If the government removes a binding price floor from a market, then the price paid by buyers will a. decrease, and the quantity sold in the market will decrease. b. decrease, and the quantity sold in the market will increase. c. increase, and the quantity sold in the market will increase. d. increase, and the quantity sold in the market will decrease.

b. decrease, and the quantity sold in the market will increase.

There are several criticisms of the minimum wage. Which of the following is not one of those criticisms? The minimum wage a. results in an excess supply of low-skilled labor. b. fails to raise the wage of any employed person. c. prevents some unskilled workers from getting needed on-the-job training. d. often hurts those people who it is intended to help.

b. fails to raise the wage of any employed person.

The goal of rent control is to a. help landlords by assuring them a low vacancy rate for their apartments. b. help the poor by making housing more affordable. c. facilitate controlled economic experiments in urban areas. d. help the poor by assuring them an adequate supply of apartments.

b. help the poor by making housing more affordable.

Over time, housing shortages caused by rent control a. increase, because the demand for and supply of housing are less elastic in the long run. b. increase, because the demand for and supply of housing are more elastic in the long run. c. decrease, because the demand for and supply of housing are less elastic in the long run. d. decrease, because the demand for and supply of housing are more elastic in the long run.

b. increase, because the demand for and supply of housing are more elastic in the long run.

Suppose researchers at the University of Wisconsin discover a new vitamin that increases the milk production of dairy cows. If the demand for milk is relatively inelastic, the discovery will a. raise both price and total revenues. b. lower both price and total revenues. c. raise price and lower total revenues. d. lower price and raise total revenues.

b. lower both price and total revenues.

If Max experiences a decrease in his income, then we would expect Max's demand for a. each good he purchases to remain unchanged. b. normal goods to decrease. c. inferior goods to decrease. d. luxury goods to increase.

b. normal goods to decrease.

Suppose that 50 hot dogs are demanded at a particular price. If the price of hot dogs rises from that price by 5 percent, the number of hot dogs demanded falls to 48. Using the midpoint approach to calculate the price elasticity of demand, it follows that the a. price increase will decrease the total revenue of hot dog sellers. b. price elasticity of demand for hot dogs in this price range is about 0.82. c. price elasticity of demand for hot dogs in this price range is about 1.22. d. demand for hot dogs in this price range is unit elastic.

b. price elasticity of demand for hot dogs in this price range is about 0.82.

If, at the current price, there is a surplus of a good, then a. the market must be in equilibrium. b. sellers are producing more than buyers wish to buy. c. the price is below the equilibrium price. d. quantity demanded equals quantity supplied.

b. sellers are producing more than buyers wish to buy.

If the price of walnuts rises, many people would switch from consuming walnuts to consuming pecans. But if the price of salt rises, people would have difficulty purchasing something to use in its place. These examples illustrate the importance of a. a necessity versus a luxury in determining the price elasticity of demand. b. the availability of close substitutes in determining the price elasticity of demand. c. the definition of a market in determining the price elasticity of demand. d. the time horizon in determining the price elasticity of demand.

b. the availability of close substitutes in determining the price elasticity of demand.

Demand is said to be inelastic if a. demand shifts only slightly when the price of the good changes. b. the quantity demanded changes only slightly when the price of the good changes. c. buyers respond substantially to changes in the price of the good. d. the price of the good responds only slightly to changes in demand.

b. the quantity demanded changes only slightly when the price of the good changes.

If consumers view cappuccinos and lattés as substitutes, what would happen to the equilibrium price and quantity of lattés if the price of cappuccinos rises? a. The equilibrium price would increase, and the equilibrium quantity would decrease. b. Both the equilibrium price and quantity would decrease. c. Both the equilibrium price and quantity would increase. d. The equilibrium price would decrease, and the equilibrium quantity would increase.

c. Both the equilibrium price and quantity would increase.

In competitive markets, which of the following is not correct? a. No individual buyer can influence the market price. b. Firms produce identical products. c. Some sellers can set prices. d. Buyers are price takers.

c. Some sellers can set prices.

For a particular good, a 5 percent increase in price causes a 15 percent decrease in quantity demanded. Which of the following statements is most likely applicable to this good? a. The market for the good is broadly defined. b. The relevant time horizon is short. c. There are many substitutes for this good. d. The good is a necessity.

c. There are many substitutes for this good.

Suppose that Jane enjoys Diet Coke so much that she consumes one can every day. Although she enjoys gourmet cheese, she consumes it sporadically. If the price of Diet Coke rises, Jane decreases her consumption by only a very small amount. But if the price of gourmet cheese rises, Jane decreases her consumption by a lot. These examples illustrate the importance of a. the time horizon in determining the price elasticity of demand. b. the definition of a market in determining the price elasticity of demand. c. a necessity versus a luxury in determining the price elasticity of demand. d. the availability of close substitutes in determining the price elasticity of demand.

c. a necessity versus a luxury in determining the price elasticity of demand.

Which of the following would shift the demand curve for gasoline to the right? a. a decrease in the price of gasoline b. a decrease in the expected future price of gasoline c. an increase in consumer income, assuming gasoline is a normal good d. an increase in the price of cars, a complement for gasoline

c. an increase in consumer income, assuming gasoline is a normal good

Under rent control, bribery is a mechanism to a. allocate housing to the most deserving tenants. b. force the total price of an apartment (including the bribe) to be less than the market price. c. bring the total price of an apartment (including the bribe) closer to the equilibrium price. d. allocate housing to the poorest individuals in the market.

c. bring the total price of an apartment (including the bribe) closer to the equilibrium price.

The discovery of a new hybrid wheat would increase the supply of wheat. As a result, wheat farmers would realize an increase in total revenue if the a. supply of wheat is elastic. b. demand for wheat is inelastic. c. demand for wheat is elastic. d. supply of wheat is inelastic.

c. demand for wheat is elastic

Recent forest fires in the western states are expected to cause the price of lumber to rise in the next six months. As a result, we can expect the supply of lumber to a. fall in six months but not now. b. increase now to meet as much demand as possible. c. fall now. d. increase in six months when the price goes up.

c. fall now.

Holding all other forces constant, if increasing the price of a good leads to an increase in total revenue, then the demand for the good must be a. unit elastic. b. elastic. c. inelastic. d. None of the above is correct because a price increase always leads to an increase in total revenue.

c. inelastic.

Suppose that a tax is placed on books. If the sellers pay the majority of the tax, then we know that the a. government has required that buyers remit the tax payments. b. demand is more inelastic than the supply. c. supply is more inelastic than the demand. d. government has required that sellers remit the tax payments

c. supply is more inelastic than the demand.

Suppose that when the price of ginger ale is $2 per bottle, firms can sell 4 million bottles. When the price of ginger ale is $3 per bottle, firms can sell 2 million bottles. Which of the following statements is true? a. The demand for ginger ale is income inelastic, so an increase in the price of ginger ale will increase the total revenue of ginger ale producers. b. The demand for ginger ale is price inelastic, so an increase in the price of ginger ale will increase the total revenue of ginger ale producers. c. The demand for ginger ale is income elastic, so an increase in the price of ginger ale will increase the total revenue of ginger ale producers. d. The demand for ginger ale is price elastic, so an increase in the price of ginger ale will decrease the total revenue of ginger ale producers.

d. The demand for ginger ale is price elastic, so an increase in the price of ginger ale will decrease the total revenue of ginger ale producers.

Suppose the government has imposed a price floor on cellular phones. Which of the following events could transform the price floor from one that is binding to one that is not binding? a. Firms expect the price of cellular phones to fall in the future. b. The components used to produce cellular phones become less expensive. c. Cellular phones become less popular. d. Traditional land line phones become more expensive.

d. Traditional land line phones become more expensive.

Suppose the demand for macaroni is inelastic, the supply of macaroni is elastic, the demand for cigarettes is inelastic, and the supply of cigarettes is elastic. If a tax were levied on the sellers of both of these commodities, we would expect that the burden of a. the macaroni tax would fall more heavily on the sellers than on the buyers, and the burden of the cigarette tax would fall more heavily on the buyers than on the sellers. b. both taxes would fall more heavily on the sellers than on the buyers. c. the macaroni tax would fall more heavily on the buyers than on the sellers, and the burden of the cigarette tax would fall more heavily on the sellers than on the buyers. d. both taxes would fall more heavily on the buyers than on the sellers.

d. both taxes would fall more heavily on the buyers than on the sellers.

When the price of a good is lower than the equilibrium price, a. quantity supplied exceeds quantity demanded. b. sellers desire to produce and sell more than buyers wish to purchase. c. a surplus will exist. d. buyers desire to purchase more than is produced.

d. buyers desire to purchase more than is produced.

The supply curve for milk a. does not shift when the price of milk changes because the price of milk is measured on the horizontal axis of the graph. b. shifts when the price of milk changes because the quantity supplied of milk is measured on the horizontal axis of the graph. c. shifts when the price of milk changes because the price of milk is measured on the vertical axis of the graph. d. does not shift when the price of milk changes because the price of milk is measured on the vertical axis of the graph.

d. does not shift when the price of milk changes because the price of milk is measured on the vertical axis of the graph.

In the housing market, supply and demand are a. more elastic in the short run than in the long run, and so rent control leads to a larger shortage of apartments in the short run than in the long run. b. more elastic in the long run than in the short run, and so rent control leads to a larger shortage of apartments in the short run than in the long run. c. more elastic in the short run than in the long run, and so rent control leads to a larger shortage of apartments in the long run than in the short run. d. more elastic in the long run than in the short run, and so rent control leads to a larger shortage of apartments in the long run than in the short run.

d. more elastic in the long run than in the short run, and so rent control leads to a larger shortage of apartments in the long run than in the short run.

If the government removes a $1 tax on sellers of gasoline and imposes the same $1 tax on buyers of gasoline, then the price paid by buyers will a. increase, and the price received by sellers will increase. b. increase, and the price received by sellers will not change. c. not change, and the price received by sellers will increase. d. not change, and the price received by sellers will not change.

d. not change, and the price received by sellers will not change.

When all market participants are price takers who have no influence over prices, the markets have a. numerous buyers but only a few sellers. b. numerous sellers but only a few buyers. c. only a few buyers and sellers. d. numerous buyers and sellers.

d. numerous buyers and sellers.

In a competitive market free of government regulation, a. price adjusts until quantity demanded is greater than quantity supplied. b. price adjusts until quantity demanded is less than quantity supplied. c. supply adjusts to meet demand at every price. d. price adjusts until quantity demanded equals quantity supplied.

d. price adjusts until quantity demanded equals quantity supplied.

Suppose roses are currently selling for $40 per dozen, but the equilibrium price of roses is $30 per dozen. We would expect a a. shortage to exist and the market price of roses to increase. b. shortage to exist and the market price of roses to decrease. c. surplus to exist and the market price of roses to increase. d. surplus to exist and the market price of roses to decrease.

d. surplus to exist and the market price of roses to decrease.

Which of the following will cause no change in producer surplus? a. the price of a substitute increases b. buyers expect the price of a good to be higher next month c. income increases and buyers consider the good to be inferior d. the imposition of a nonbinding price ceiling in the market

d. the imposition of a nonbinding price ceiling in the market

Generally, a firm is more willing and able to increase quantity supplied in response to a price change when a. the relevant time period is short rather than long. b. the firm is experiencing capacity problems. c. supply is inelastic. d. the relevant time period is long rather than short.

d. the relevant time period is long rather than short.

The marginal seller is the seller a. for whom the marginal cost of producing one more unit of output is the lowest among all sellers, and the marginal buyer is the buyer for whom the marginal benefit of one more unit of the good is the highest among all buyers. b. who supplies the smallest quantity of the good among all sellers, and the marginal buyer is the buyer who demands the smallest quantity of the good among all buyers. c. who has the largest producer surplus, and the marginal buyer is the buyer who has the largest consumer surplus. d. who would leave the market first if the price were any lower, and the marginal buyer is the buyer who would leave the market first if the price were any higher.

d. who would leave the market first if the price were any lower, and the marginal buyer is the buyer who would leave the market first if the price were any higher.


Kaugnay na mga set ng pag-aaral

Quiz: Module 07 Network Architecture

View Set

Cells and Layers of The Epidermis

View Set

LAB EXERCISE 9.2 Determining Elevations from Topographic Maps

View Set